حمید آنالیز

Well-Known Member
ارسال ها
1,351
لایک ها
1,322
امتیاز
113
پاسخ : ماراتن جبر (سطح پیشرفته)

فکر می کنم به این سوال جواب داده نشده باشد.

ابتدا به این نکته توجه کنید که تابع
یک به یک و پوشاست. از نکته بعدا در اثبات استفاده می شود.

داریم:
از تقارن عبارت و یک به یکی تابع نتیجه می گیریم که برای هر دو عدد حقیقی و مثبت
داریم:


ادعا می کنیم
. فرض کنید این طور نباشد. در این صورت دو حالت داریم:
که در تناقض با پوشایی
است.
که این نیز در تناقض با پوشایی
است.

پس
. اکنون از عبارت مساله داریم:

پس


بنابراین:

پوشایی نتیجه می دهد که تابع
جمعی است.

از اینجا با توجه به برد تابع
نتیجه می شود که:

در فرض مساله این تابع را جایگذاری می کنیم و نتیجه می شود که:

D:
خیلی ممنون از حلتون لطفا سوال بعد رو لطف کنین.با تشکر
 

Dadgarnia

New Member
ارسال ها
1,350
لایک ها
1,127
امتیاز
0
پاسخ : ماراتن جبر (سطح پیشرفته)

فکر می کنم به این سوال جواب داده نشده باشد.

ابتدا به این نکته توجه کنید که تابع
یک به یک و پوشاست. از نکته بعدا در اثبات استفاده می شود.

داریم:
از تقارن عبارت و یک به یکی تابع نتیجه می گیریم که برای هر دو عدد حقیقی و مثبت
داریم:


ادعا می کنیم
. فرض کنید این طور نباشد. در این صورت دو حالت داریم:
که در تناقض با پوشایی
است.
که این نیز در تناقض با پوشایی
است.

پس
. اکنون از عبارت مساله داریم:

پس


بنابراین:

پوشایی نتیجه می دهد که تابع
جمعی است.

از اینجا با توجه به برد تابع
نتیجه می شود که:

در فرض مساله این تابع را جایگذاری می کنیم و نتیجه می شود که:

D:
ببخشيد فكر كنم سوالو اشتباه گذاشته بودم. البته اين سوالي كه من به اشتباه گذاشتم هم سوال خوبي بود براي فكر كردن. سوالي كه من مي خواستم بذارم دامنه و برد تابعش اعداد صحيح مثبت بود.
 

Amitis :D

New Member
ارسال ها
21
لایک ها
15
امتیاز
3
پاسخ : ماراتن جبر (سطح پیشرفته)

ببخشيد فكر كنم سوالو اشتباه گذاشته بودم. البته اين سوالي كه من به اشتباه گذاشتم هم سوال خوبي بود براي فكر كردن. سوالي كه من مي خواستم بذارم دامنه و برد تابعش اعداد صحيح مثبت بود.
راه حل برای وقتی که دامنه و برد تابع اعداد صحیح مثبت هست:

مجددا دیده می شود که تابع یک به یک است. بنابراین برای هر 4 عدد صحیح مثبت
که
داریم:



حالا اتحاد لاگرانژ رو یادآوری می کنم:

بنابراین برای هر 4 عدد صحیح و مثبت
که
داریم:


قرار بدید:

به دست می آید:
برای

بنابراین برای هر عدد صحیح و مثبت
داریم:

پس دنباله ی
یک دنباله ی بازگشتی خطی است که معادله ی مشخصه ی آن

بنابراین:

این جواب رو اگر در معادله ی اصلی مساله جایگذاری کنید به دست میاد:
(*)


تعریف کنید:


حالا اگر طرفین (*) رو به توان دو برسونیم خواهیم داشت:

حالا مقایسه ی ضرایب رو انجام میدیم:


بنابراین:

اما می دونیم تفاضل دو عدد مربع کامل نابرابر، بی کران است. بنابراین نتیجه می شود که:

D:
 
آخرین ویرایش توسط مدیر

J.Karimi

Active Member
ارسال ها
169
لایک ها
115
امتیاز
43
پاسخ : ماراتن جبر (سطح پیشرفته)

اگر
, و اعداد ما حقيقي و مثبت باشند ثابت كنيد:

 
آخرین ویرایش توسط مدیر

Dadgarnia

New Member
ارسال ها
1,350
لایک ها
1,127
امتیاز
0
پاسخ : ماراتن جبر (سطح پیشرفته)

اگر
, و اعداد ما حقيقي و مثبت باشند ثابت كنيد:

با استفاده از نامساوي
بايد ثابت كنيم:
با استفاده از كوشي داريم:

با استفاده از شور كافيه ثابت كنيم:

پس حكم ثابت شد.
 

J.Karimi

Active Member
ارسال ها
169
لایک ها
115
امتیاز
43
پاسخ : ماراتن جبر (سطح پیشرفته)

با استفاده از نامساوي
بايد ثابت كنيم:
با استفاده از كوشي داريم:

با استفاده از شور كافيه ثابت كنيم:

پس حكم ثابت شد.
باتشكر فراوان از آقاي دادگرنيا

اين سوال رو يكي از دوستانم گفت كه توي تمرينات هولدر يه كتابي مثل اينكه ديده ببينيد مي تونيد اين رو با هولدر هم اثبات كنيد

من تا اينجا رسوندم ولي نتونستن ادامه اش رو تكميل كنم:





درنتيجه بايد ثابت كنيم:



ولي هركاري كردم نتونستم قشنگ جور در بيارمش ممنون ميشم اگه بتونيد كاملش كنيد!!!
 

Dadgarnia

New Member
ارسال ها
1,350
لایک ها
1,127
امتیاز
0
پاسخ : ماراتن جبر (سطح پیشرفته)

باتشكر فراوان از آقاي دادگرنيا

اين سوال رو يكي از دوستانم گفت كه توي تمرينات هولدر يه كتابي مثل اينكه ديده ببينيد مي تونيد اين رو با هولدر هم اثبات كنيد

من تا اينجا رسوندم ولي نتونستن ادامه اش رو تكميل كنم:





درنتيجه بايد ثابت كنيم:



ولي هركاري كردم نتونستم قشنگ جور در بيارمش ممنون ميشم اگه بتونيد كاملش كنيد!!!
با استفاده از نامساوي
(توي راه حل من هم اگه به جاي شور از اين نامساوي استفاده مي كردم سوال اثبات ميشد) كافيه ثابت كنيم:
اين هم واضحه.

 

Dadgarnia

New Member
ارسال ها
1,350
لایک ها
1,127
امتیاز
0
پاسخ : ماراتن جبر (سطح پیشرفته)

سوال بعد:
تمام چند جمله اي هاي
را بيابيد به طوريكه
.
 
آخرین ویرایش توسط مدیر
ارسال ها
288
لایک ها
154
امتیاز
43
پاسخ : ماراتن جبر (سطح پیشرفته)

بعضياشو مربع ميذاره بازم نميكنه امشب خرابه فك كنم

---- دو نوشته به هم متصل شده است ----

بعضياشو مربع ميذاره بازم نميكنه امشب خرابه فك كنم
 
ارسال ها
288
لایک ها
154
امتیاز
43
پاسخ : ماراتن جبر (سطح پیشرفته)

جوابش ميشه. 1= (p(x و يا 0=(p(x
اگه درسته بگيد راه حلمو بذارم
 

Sharifi_M

New Member
ارسال ها
561
لایک ها
348
امتیاز
0
پاسخ : ماراتن جبر (سطح پیشرفته)

سوال بعد:
تمام چند جمله اي هاي
را بيابيد به طوريكه
.




معادله آخر دو تا جواب ثابت داره:
و

بعد اگه ثابت نباشه میشه اینجوری نوشت این چند جمله ای رو:

بعد اگر اینو تو معادله بالامون بزاریم و ضرایب جملات
رو تو دو طرف تساوی مقایسه کنیم و استقرا بزنیم نتیجه میشه که:
پس مجموعه جواب های
شد
پس برای
خواهیم داشت


پی‌نوشت: معادله
رو قبل ها مستقل از این مسئله حل کرده بودم و فقط کلیتش یادم بود و جواب نهایی! :4: و حوصله باز کردن نداشتم! به بزرگی خودتون ببخشید!

---- دو نوشته به هم متصل شده است ----

سوال بعد:

و

ماکزیمم عبارت زیر را بیابید:
 

Dadgarnia

New Member
ارسال ها
1,350
لایک ها
1,127
امتیاز
0
پاسخ : ماراتن جبر (سطح پیشرفته)

اول واضحه که می تونیم فرض کنیم
نامنفی اند. با استفاده از حسابی هندسی داریم:
با استفاده از اتحاد لاگرانژ و حسابی هندسی داریم:



حالت تساوی هم برای
اتفاق می افته.

---- دو نوشته به هم متصل شده است ----

سوال بعد:
برای چند جمله ای
داریم
و قرار می دهیم
. اگر
دو ریشه از ریشه های
باشند ثابت کنید
ریشه ای مثبت و کوچکتر از
دارد.
 
آخرین ویرایش توسط مدیر

Dadgarnia

New Member
ارسال ها
1,350
لایک ها
1,127
امتیاز
0
پاسخ : ماراتن جبر (سطح پیشرفته)

راهنمایی: ثابت کنید
.
 
ارسال ها
288
لایک ها
154
امتیاز
43
پاسخ : ماراتن جبر (سطح پیشرفته)

اصلا هيچ كدوم عبارتا را باز نميكنه لاتكس مشكل پيدا كرده
 
آخرین ویرایش توسط مدیر

Dadgarnia

New Member
ارسال ها
1,350
لایک ها
1,127
امتیاز
0
پاسخ : ماراتن جبر (سطح پیشرفته)

براي من كه درسته چند بار ديگه سعي كنيد شايد مشكل از طرف شما باشه.
 
ارسال ها
288
لایک ها
154
امتیاز
43
پاسخ : ماراتن جبر (سطح پیشرفته)

به علت اين كه سوال قبل به مدت زيادي حل نشده باقي موند اين سوالو گذاشتم اگه كسي تونست سوال قبلي را هم حلشو بذاره
و
چند جمله اي هايي با ضرايب حقيقي و از درجه ي ٢ هستند. ثابت كنيد اگر بدانيم
و
هيچ كدام ريشه ي حقيقي ندارند . آنگاه حداقل يكي از معادلات
و
هم جواب ندارند.
 
آخرین ویرایش توسط مدیر

Sharifi_M

New Member
ارسال ها
561
لایک ها
348
امتیاز
0
پاسخ : ماراتن جبر (سطح پیشرفته)

سوال بعد:
برای چند جمله ای
داریم
و قرار می دهیم
. اگر
دو ریشه از ریشه های
باشند ثابت کنید
ریشه ای مثبت و کوچکتر از
دارد.
بعد از مدت طولانی دوباره به فکرش افتادم :4: ، داریم:



...
پس داریم:


حالا اگر
ریشه ای در این بین نداشته باشد، آنگاه مقدار
ها باید هم علامت باشند (در صورت نبود از قضیه مقدار میانی میشه فهمید ریشه ای وجود داره) که با فرض اینکه مجموعشون صفره در تناقضه پس فرض اولیه ما غلطه و
ریشه ای در
تا
داره.
اگر صفر شود ریشه ی دیگری بین
تا
وجود خواهد داشت(مجموع P هایشان صفر خواهد شد.) و در مجموع نتیجه میگیریم که P ریشه ای کوچکتر از
و مثبت دارد.
 

Dadgarnia

New Member
ارسال ها
1,350
لایک ها
1,127
امتیاز
0
پاسخ : ماراتن جبر (سطح پیشرفته)

به علت اين كه سوال قبل به مدت زيادي حل نشده باقي موند اين سوالو گذاشتم اگه كسي تونست سوال قبلي را هم حلشو بذاره
و
چند جمله اي هايي با ضرايب حقيقي و از درجه ي ٢ هستند. ثابت كنيد اگر بدانيم
و
هيچ كدام ريشه ي حقيقي ندارند . آنگاه حداقل يكي از معادلات
و
هم جواب ندارند.
https://www.artofproblemsolving.com/community/c6h147156p832720

---- دو نوشته به هم متصل شده است ----

بعد از مدت طولانی دوباره به فکرش افتادم :4: ، داریم:



...
پس داریم:


حالا اگر
ریشه ای در این بین نداشته باشد، آنگاه مقدار
ها باید هم علامت باشند (در صورت نبود از قضیه مقدار میانی میشه فهمید ریشه ای وجود داره) که با فرض اینکه مجموعشون صفره در تناقضه پس فرض اولیه ما غلطه و
ریشه ای در
تا
داره.
اگر صفر شود ریشه ی دیگری بین
تا
وجود خواهد داشت(مجموع P هایشان صفر خواهد شد.) و در مجموع نتیجه میگیریم که P ریشه ای کوچکتر از
و مثبت دارد.
اینکه مجموعشون صفره رو هم ثابت کن بعدش هم سوال بعدی رو بذار.
 
بالا